Pular para o conteúdo principal

Pesquise aqui

Na figura ao lado, a resta do cubo maior mede $a$, e os outros cubos foram construídos de modo que a medida... Resposta comentada

Infinitos cubos 

U. E. Londrina PR | Na figura ao
 lado, a aresta do cubo maior mede $a$, e os outros cubos foram construídos de modo que a medida da respectiva aresta seja a metade da aresta do cubo anterior. Imaginando que a construção continue indefinidamente, a soma dos volumes de todos os cubos será: 

a) $0$       b) $\frac{a^{3}}{2}$         c) $\frac{7a^{3}}{8}$        d)$\frac{8a^{3}}{7}$       e)$2a^{^{3}}$




Solução:

Encontrando os volumes dos primeiros cubos, vamos ter:

1º $a^{^{3}}$
2º $\left ( \frac{a}{2} \right )^{3} = \frac{a^{3}}{8}$
3º $\left ( \frac{a}{4} \right )^{3} = \frac{a^{3}}{64}$
4º $\left ( \frac{a}{8} \right )^{3} = \frac{a^{3}}{512}$
.
.
.

Percebemos que se trata de uma P. G. decrescente. Para encontrar o volume pedido, iremos recorrer a fórmula da soma de uma P.G. infinita que é $S_{\infty }=\frac{a_{1}}{1 -q}$, onde $a_{1} = a^{3}$ Mas antes precisamos encontrar a razão $q$. Para isto iremos dividir o segundo termo pelo primeiro. Assim, temos:


$q =\frac{\frac{a^{3}}{8}}{a^{3}} = \frac{a^{3}}{8}\cdot \frac{1}{a^{3}}=\frac{1}{8} \rightarrow q = \frac{1}{8}$ Logo:

$S_{\infty }=\frac{a_{1}}{1 -q}= \frac{a^{3}}{1-\frac{1}{8}} = \frac{a^{3}}{\frac{7}{8}}= a^{3}\cdot \frac{8}{7}=\frac{8a^{3}}{7}$

Resposta: A soma dos volumes de todos os cubos é $\frac{8a^{3}}{7}$ letra d)





Comentários